You are on page 1of 4

Homework 2 Solutions

1. (Exercise 7.2)

Let {fn } and {gn } converge uniformly on a set E .

Claim. {fn + gn } converges uniformly on E . If, in addition, {fn } and {gn } are sequences of bounded
functions, then {fn gn } converges uniformly on E .
Proof. Let f, g denote the limits of {fn }, {gn } respectively. Let  > 0. Find Nf such that n Nf =
|fn (x) f (x)| < 2 on E and nd Ng such that n Ng = |gn (x) g(x)| < 2 on E . Let N = max{Nf , Ng }.
Then, if n N and x E , we have |(fn + gn )(x) (f + g)(x)| |fn (x) f (x)| + |gn (x) g(x)| < . Hence,
fn + gn converges uniformly to f + g .

Now, suppose {fn }, {gn } are sequences of bounded functions. We rst show there are uniform bounds
Mf , Mg such that |fn (x)| Mf and |gn (x)| Mg for all n Z+ , x E . We do this for {fn } and the
proof for {gn } will be the same. Let  = 1 and nd N such that x E, n N = |fn (x) f (x)| < 1.
Let M1 , . . . MN be bounds for f1 , . . . , fN (ie |fk (x)| Mk for all x E ). Now, if xE and n N , we see
|fn (x) fN (x)| |fn (x) f (x)| + |f (x) fN (x)| 2, so |fn (x)| MN + 2 for all x E and n N . Thus,
Mf = max{M1 , . . . , MN 1 , MN + 2} is a uniform upper bound. Let Mg be the simialr bound found for {gn }.
Let M = max{Mf , Mg }.
We show {fn gn } converges uniformly to f g . Indeed, let  > 0. Let N be such that if x E and n N ,


and |gn (x) g(x)| < 2M
. We have, for such x, n,
we have |fn (x) f (x)| < 2M
|fn (x)gn (x) f (x)g(x)|

|fn (x)gn (x) f (x)gn (x) + f (x)gn (x) f (x)g(x)|

|fn (x) f (x)||gn (x)| + |f (x)||gn (x) g(x)|




M +M
<
2M
2M
= 

which proves the claim.


2. (Exercise 7.3) Construct uniformly convergent {fn }, {gn } on a set E such that fn gn h for some
function h but not uniformly.

Take E = R, fn (x) = x, gn (x) = n1 . Then, {fn } converges uniformly to f (x) = x, {gn }


converges uniformly to 0, and fn gn (x) = nx 0 for every x R, but for  = 1, there is an xn = n such that
fn gn (xn ) = nn = 1 and |1 0| = 1 , so fn gn does not converge uniformly.
Example.

3. (Exercise 7.5)

Let

(
1
sin2 ( x ) n+1
x
fn (x) =
0
otherwise

1
n

converges to a continuous function, but not uniformly. Moreover,


for all x, but not uniformly.
Claim. {fn }

fn coverges absolutely

1
For x (0, 1), we see fn (x) = 0 for all n such that x / [ n+1
, n1 ] and for x
/ (0, 1), we see
2
2
2
fn (x) = 0 for all n, so fn 0. To see this is not uniform, let  = 1. Then, 2(n+1)
2n+1
2n
and so

Proof.

fn (

1
) = sin2 ( 2 ) = sin2 ((n + )) = 1
2n + 1
2
2n+1

so for each n there is an x (0, 1) such that |fn (x) f (x)| = 1 .


1

Now, we see fn 0 for all n and in


absolutely to f given by

fn , for xed x at most one term is nonzero, so this converges

(
sin2 ( x ) 0 < x 1
f (x) =
0
otherwise

This sum does not converge uniformly, however, as for any N Z+ , we see
|

N
X

fn (

n=1

2
2

) f(
)| = |0 sin2 ((N + 1) + )| = 1
2(N + 1) + 1
2(N + 1) + 1
2

4. (Exercise 7.6) Claim.

The series

(1)n

n=1

x2 + n
n2

converges uniformly in every bounded interval, but does not converge absolutely for any value of x.
Proof. It suces to show uniform convergence on intervals of the form [a, a] where a > 0. By the
alternating series test, we have convergence to a limit function f on R. Moreover, by the proof of that test,
we have |f (x) fn (x)| |fn+1 (x) fn (x)| for each x R and n Z+ . We have

|fn+1 (x) fn (x)| =

and, on [a, a], we have


2(

as n .

x2 + n
x2 + n x2 + n + 1
+

2(
)
n2
(n + 1)2
n2

x2 + n
a2 + n
) 2(
)0
2
n
n2

Now, this series does not converge absolutely for any x as, for N Z+
N
N
N
X
X
x2 + n X x2
1
1
=
+

2
2
n
n
n n=1 n
n=1
n=1

and the harmonic series diverges.


5. (Exercise 7.7)

For n Z+ , x R, put
fn (x) =

x
1 + nx2

converges uniformly to a function f , and f 0 (x) = limn fn0 (x) is correct for x 6= 0, but this
equation is false for x = 0.
Claim. {fn }

Proof. Note for xed x R, fn (x) 0 as n . To show uniform convergence, we need to show
supxR |fn (x)| 0 as n . We see lim|x| |fn (x)| = 0. As fn is dierentiable everywhere, we look for

critical points. Dierentiating, we have

fn0 (x) =

(1 + nx2 ) x2nx
1 nx2
=
(1 + nx2 )2
(1 + nx2 )2

which is 0 at x = 1n . We have
1

1
1
n
fn ( ) =
=
n
2 n
1 + n( 1n )2

1
) = fn ( 1n ), so supxR |fn (x)| =
and fn (
n

2 n

0 as n . That is, the convergence is uniform.

We now examine fn0 (x). For x R, x 6= 0, we have


fn0 (x) =

1 nx2
(1 + nx2 )2

and so, for n such that nx2 2, we have


|fn0 (x)|

nx2
1
=
0
(nx2 )2
nx2

as n , so we have f 0 (x) = limn fn0 (x) for x 6= 0. At 0, however, we have fn0 (0) = 1 for all n, so
fn0 (0) 1 6= 0.
6.

Let fn : R R be dierentiable for each n and be such that |fn0 (x)| 1 for all n, x. Suppose fn f .

Claim. f

is continuous.

Proof. We show for x, y R, |f (x) f (y)| |x y|, which shows the result (take =  in the denition
of continuity at x). Fix x, y R with x 6= y . We assume y > x, and the case y < x is similar (replace (x, y)
with (y, x) in what follows). Then, by the mean value theorem, there is a t with t (x, y) such that

fn (x) fn (y)
= fn0 (t)
xy

Thus, we have |fn (x) fn (y)| |fn0 (t)||x y| |x y|. Let  > 0 and let N be such that n N =
|fn (x) f (x)| <  and |fn (y) f (y)| < . Then,
|f (x) f (y)| |f (x) fn (x)| + |fn (x) fn (y)| + |fn (y) f (y)| < 2 + |x y|

As  > 0 was arbitrary, we conclude


|f (x) f (y)| |x y|

7.

Let fn (x) =

sin(nx)

(a) Claim. {fn }


Proof.

converges to 0 uniformly.

For all x we have |fn (x)|

(b) Claim.
Proof.

1
n

, so supxR |fn (x)|

1
n

0 as n .

For any x R, the sequence of derivatives fn0 (x) diverges.

We have fn0 (x) = n cos(nx). Fix x R.

We work mod . That is, for every y R, there is a unique integer m Z such that y mx [0, ).
We use the notation y mod for y mx. To keep the notation clean, we use [y] to denote y mod . Note
that [x] + [y] = [x + y] and | cos(y)| = | cos([y])| as | cos | is -periodic.
Consider the sequence [nx] where n Z+ . Suppose [nx] = [mx] for some n > m > 0. Then,
[(n m)x] = 0. Thus, if ` = n m > 0, we have | cos(kx + `x)| = | cos(kx)| for all k Z+ . Now, we
claim one of | cos(x)|, | cos(2x)| is nonzero. To see this, if | cos(x)| = 0, then x = (p + 21 ) for some integer
p, so cos(2x) = cos((2p +
of | cos((1 + k)x)| or | cos((2 + k)x)| is a constant
p1)) = 1. Hence, one p
nonzero
sequence, and so one of (1 + k) cos((1 + k)x) or (2 + k) cos((2 + k)x) diverges. In either case, n cos(nx)
diverges.
3

Now, suppose [nx] 6= [mx] when n 6= m for n, m Z+ . We claim {[nx]} is dense in (0, ). Indeed, let
m Z+ and consider partitioning [0, ] into m equilength intervals. By the pigeonhole principle, two of the

]
numbers [x], . . . , [(m+1)x] must lie in the same interval, say [kx] and [`x] with ` > k. Thus, [(`k)x] [0, m

+
and the set {[p(` k)x] : p Z } {[nx]} is m -dense in [0, ). As m was arbitrary, we see that {[nx]} is
dense in [0, ). Now, let A [0, ) be an open set such that if a A, then | cos(a)| > 21 . This exists as cos
is continuous and cos(0) = 1. As {[nx]} is dense in [0, ), there are innitely many
elements of {[nx]} in

1
A
.
That
is,
there
is
a
subsequence
n
such
that
|
cos(n
x)|
>
for
all
k
.
Thus,
n
|
cos(n
k
k
k
k x)| diverges, so
2

n cos(nx) diverges.

You might also like